上一主题:Reading 49: The Asset Allocation Decision - LOS c ~ Q1-3
下一主题:Reading 50: An Introduction to Portfolio Management - LOS
返回列表 发帖

Reading 50: An Introduction to Portfolio Management - LOS

Q7. The optimal portfolio in the Markowitz framework occurs when an investor achieves the diversified portfolio with the:

A)   highest utility.

B)   highest return.

C)   lowest risk.

Q8. The graph below combines the efficient frontier with the indifference curves for two different investors, X and Y (represented by U(X) and U(Y)). The letters A, B, C, and D represent four distinct portfolios.

Which of the following statements about the above graph is least accurate?

A)   Investor X's return will always be less than that of Investor Y.

B)   Investor X would be better off moving to indifference curve U(X)1 and Portfolio C because of the higher return on that portfolio.

C)   Portfolios A and B are both optimal portfolios.

Q9. Which of the following statements about the optimal portfolio is FALSE? The optimal portfolio:

A)   lies at the point of tangency between the efficient frontier and the indifference curve with the highest possible utility.

B)   is the portfolio that gives the investor the maximum level of return.

C)   may be different for different investors.

Q10. Which of the following statements about the efficient frontier is least accurate?

A)   Portfolios falling on the efficient frontier are fully diversified.

B)   The efficient frontier shows the relationship that exists between expected return and total risk in the absence of a risk-free asset.

C)   Investors will want to invest in the portfolio on the efficient frontier that offers the highest rate of return.

答案和详解如下:

Q7. The optimal portfolio in the Markowitz framework occurs when an investor achieves the diversified portfolio with the:

A)   highest utility.

B)   highest return.

C)   lowest risk.

Correct answer is A)

The optimal portfolio in the Markowitz framework occurs when the investor achieves the diversified portfolio with the highest utility.

Q8. The graph below combines the efficient frontier with the indifference curves for two different investors, X and Y (represented by U(X) and U(Y)). The letters A, B, C, and D represent four distinct portfolios.

Which of the following statements about the above graph is least accurate?

A)   Investor X's return will always be less than that of Investor Y.

B)   Investor X would be better off moving to indifference curve U(X)1 and Portfolio C because of the higher return on that portfolio.

C)   Portfolios A and B are both optimal portfolios.

Correct answer is B)

Any portfolio on the efficient frontier is superior to one that is not. Thus, Investor X would not be better off with Portfolio C (this portfolio is on a lower indifference curve and has more risk.)

The other choices are correct. The optimal portfolio for each investor is the one on the highest indifference curve that is tangent to the efficient frontier. Thus, portfolios A and B are both optimal portfolios, but for different investors. In addition, Investor X has a steeper indifference curve, indicating that he is risk-averse. Flatter curves, such as those for investor Y, indicate a less risk-averse investor. As a result, X’s return will be less than Y’s.

Q9. Which of the following statements about the optimal portfolio is FALSE? The optimal portfolio:

A)   lies at the point of tangency between the efficient frontier and the indifference curve with the highest possible utility.

B)   is the portfolio that gives the investor the maximum level of return.

C)   may be different for different investors.

Correct answer is B)

This statement is incorrect because it does not specify that risk must also be considered.

Q10. Which of the following statements about the efficient frontier is least accurate?

A)   Portfolios falling on the efficient frontier are fully diversified.

B)   The efficient frontier shows the relationship that exists between expected return and total risk in the absence of a risk-free asset.

C)   Investors will want to invest in the portfolio on the efficient frontier that offers the highest rate of return.

Correct answer is C)

The optimal portfolio for each investor is the highest indifference curve that is tangent to the efficient frontier.

 

TOP

see

TOP

abc

TOP

Thanks

TOP

a

TOP

thanks

TOP

great

TOP

thx

TOP

[em50]

TOP

返回列表
上一主题:Reading 49: The Asset Allocation Decision - LOS c ~ Q1-3
下一主题:Reading 50: An Introduction to Portfolio Management - LOS